Difference between revisions of "2022 AMC 12B Problems/Problem 17"

(Solution (quick and easy))
(Problem)
Line 1: Line 1:
 
==Problem==
 
==Problem==
  
How many <math>4 \times 4</math> arrays whose entries are <math>0</math>s and <math>1</math>s are there such that the row sums (the sum of the
+
How many <math>4 \times 4</math> arrays whose entries are <math>0</math>s and <math>1</math>s are there such that the row sums (the sum of the entries in each row) are <math>1, 2, 3,</math> and <math>4,</math> in some order, and the column sums (the sum of the entries in each column) are also <math>1, 2, 3,</math> and <math>4,</math> in some order? For example, the array
entries in each row) are 1, 2, 3, and 4, in some order, and the column sums (the sum of the entries in
+
<cmath>\left[
each column) are also 1, 2, 3, and 4, in some order? For example, the array
 
<cmath>
 
\[
 
\left[
 
 
   \begin{array}{cccc}
 
   \begin{array}{cccc}
 
     1 & 1 & 1 & 0 \\
 
     1 & 1 & 1 & 0 \\
Line 13: Line 9:
 
     0 & 1 & 0 & 0 \\
 
     0 & 1 & 0 & 0 \\
 
   \end{array}
 
   \end{array}
\right]
+
\right]</cmath>
\]
 
</cmath>
 
 
 
 
satisfies the condition.
 
satisfies the condition.
  

Revision as of 00:08, 10 January 2023

Problem

How many $4 \times 4$ arrays whose entries are $0$s and $1$s are there such that the row sums (the sum of the entries in each row) are $1, 2, 3,$ and $4,$ in some order, and the column sums (the sum of the entries in each column) are also $1, 2, 3,$ and $4,$ in some order? For example, the array \[\left[   \begin{array}{cccc}     1 & 1 & 1 & 0 \\     0 & 1 & 1 & 0 \\     1 & 1 & 1 & 1 \\     0 & 1 & 0 & 0 \\   \end{array} \right]\] satisfies the condition.

$\textbf{(A) }144 \qquad \textbf{(B) }240 \qquad \textbf{(C) }336 \qquad \textbf{(D) }576 \qquad \textbf{(E) }624$

Solution (quick and easy)

Realize the configurations of rows and columns to obtain sums of one of each of $1, 2, 3,$ and $4$ are independent of each other. Then the answer is simply $(4!)^2=\boxed{\textbf{(D) 576}}$.

~bad_at_mathcounts

Solution (Linear transformation, permutation)

In this problem, we call a matrix that satisfies all constraints given in the problem a feasible matrix.

First, we observe that if a matrix is feasible, and we swap two rows or two columns to get a new matrix, then this new matrix is still feasible.

Therefore, any feasible matrix can be obtained through a sequence of such swapping operations from a feasible matrix where for all $i \in \left\{ 1, 2, 3 ,4 \right\}$, the sum of entries in row $i$ is $i$ and the sum of entries in column $i$ is $i$, hereafter called as a benchmark matrix.

Second, we observe that there is a unique benchmark matrix, as shown below: \[ \left[   \begin{array}{cccc}     0 & 0 & 0 & 1 \\     0 & 0 & 1 & 1 \\     0 & 1 & 1 & 1 \\     1 & 1 & 1 & 1 \\   \end{array} \right] \]

With above observations, we now count the number of feasible matrixes. We construct a feasible matrix in the following steps.

Step 1: We make a permutation of rows of the benchmark matrix.

The number of ways is $4!$.

Step 2: We make a permutation of columns of the matrix obtained after Step 1.

The number of ways is $4!$.

Following from the rule of product, the total number of feasible matrixes is $4! \cdot 4! = \boxed{\textbf{(D) 576}}$.

~Steven Chen (Professor Chen Education Palace, www.professorchenedu.com)

Alternate Solution (From outside to inside)

Since exactly $1$ row sum is $4$ and exactly $1$ column sum is $4$, there is a unique entry in the array such that it, and every other entry in the same row or column, is a $1.$ Since there are $16$ total entries in the array, there are $16$ ways to choose the entry with only $1$s in its row and column.

WLOG, let that entry be in the top-left corner of the square. Note that there is already $1$ entry numbered $1$ in each unfilled row, and $1$ entry numbered $1$ in each unfilled column. Since exactly $1$ row sum is $1$ and exactly $1$ column sum is $1$, there is a unique entry in the $3\textrm{x}3$ array of the empty squares such that it, and every other entry in the same row or column in the $3\textrm{x}3$ array is a $0.$ Using a process similar to what we used in the first paragraph, we can see that there are $9$ ways to choose the entry with only $0$ in its row and column (in the $3\textrm{x}3$ array).

WLOG, let that entry be in the bottom right corner of the square. Then, the remaining empty squares are the $4$ center squares. Of these, one of the columns of the empty $2\textrm{x}2$ array will have $2$ $1$s and the other column will have $1$ $1.$ That happens if and only if exactly $1$ of the remaining squares is filled with a $0$, and there are $4$ ways to choose that square. Filling that square with a $0$ and the other $3$ squares with $1$s completes the grid.

All in all, there are $4*9*16=\boxed{576}$ ways to complete the grid.

pianoboy. ~mathboy100 (minor LaTeX fix)

Video Solution

https://youtu.be/_dN_ZHiaiko

~Steven Chen (Professor Chen Education Palace, www.professorchenedu.com)

See Also

2022 AMC 12B (ProblemsAnswer KeyResources)
Preceded by
Problem 16
Followed by
Problem 18
1 2 3 4 5 6 7 8 9 10 11 12 13 14 15 16 17 18 19 20 21 22 23 24 25
All AMC 12 Problems and Solutions

The problems on this page are copyrighted by the Mathematical Association of America's American Mathematics Competitions. AMC logo.png